Czy ten dowód jest dobry? Nierówność sumy modułów

Permutacje. Kombinacje. Wariacje. Rozmieszczanie kul w urnach. Silnie i symbole Newtona. Przeliczanie zbiorów. Funkcje tworzące. Teoria grafów.
Awatar użytkownika
VirtualUser
Użytkownik
Użytkownik
Posty: 443
Rejestracja: 2 wrz 2017, o 11:13
Płeć: Mężczyzna
Podziękował: 113 razy
Pomógł: 15 razy

Czy ten dowód jest dobry? Nierówność sumy modułów

Post autor: VirtualUser »

Witam, mam taki dowód:
Wykaż, że \(\displaystyle{ \left| \sum_{k=1}^{n} x_k \right| \le \sum_{k+1}^{n} \left| x_k\right|}\)

Mój dowód:
Zauważmy, że dla \(\displaystyle{ n=1 \Rightarrow L=P}\)
Ponadto uznajmy, że \(\displaystyle{ L(k)}\) to wartość funkcji określonej po lewej stronie równości, zaś \(\displaystyle{ P(k)}\) wartości po prawej stronie równości.

Zauważmy ponadto, że \(\displaystyle{ P(k+1) \le P(k)}\) ponieważ dodajemy tylko nieujemne wartości (z definicji modułu)
Zauważmy ponadto, że \(\displaystyle{ L(k+1)}\) może być zarówno mniejsze/większe bądź równe \(\displaystyle{ L(k)}\) gdyż z prawa trychotomii dodajemy wartość dodatnią/ujemną/zero, zaś ta zależnie od obecnego stanu \(\displaystyle{ L(k)}\)sprawia, że \(\displaystyle{ L(k+1) >/</= L(k)}\)
Zauważmy ponadto, że w obu przypadkach operujemy na tych samych wartościach elementów ze zbioru \(\displaystyle{ X}\).
Na tej podstawie stwierdzam, że \(\displaystyle{ L(k) \le P(k)}\) zatem

\(\displaystyle{ \left| \sum_{k=1}^{n} x_k \right| \le \sum_{k+1}^{n} \left| x_k\right|}\) CND

I tutaj mam dwa pytania:
a) Czy ten dowód jest poprawny?
b) Czy jest jakiś bardziej elegancki dowód?
Awatar użytkownika
leg14
Użytkownik
Użytkownik
Posty: 3132
Rejestracja: 5 lis 2014, o 20:24
Płeć: Mężczyzna
Lokalizacja: Radom
Podziękował: 154 razy
Pomógł: 475 razy

Czy ten dowód jest dobry? Nierówność sumy modułów

Post autor: leg14 »

Nie jest poprawny. Nierówność tę zazwyczaj udowadniamy indukcyjnie - no i nie powiedziałeś co to za moduł
Awatar użytkownika
VirtualUser
Użytkownik
Użytkownik
Posty: 443
Rejestracja: 2 wrz 2017, o 11:13
Płeć: Mężczyzna
Podziękował: 113 razy
Pomógł: 15 razy

Re: Czy ten dowód jest dobry? Nierówność sumy modułów

Post autor: VirtualUser »

Miałem na myśli \(\displaystyle{ \left| x\right|}\). Dlaczego nie jest poprawny?
Jan Kraszewski
Administrator
Administrator
Posty: 34287
Rejestracja: 20 mar 2006, o 21:54
Płeć: Mężczyzna
Lokalizacja: Wrocław
Podziękował: 3 razy
Pomógł: 5203 razy

Czy ten dowód jest dobry? Nierówność sumy modułów

Post autor: Jan Kraszewski »

VirtualUser pisze:Ponadto uznajmy, że \(\displaystyle{ L(k)}\) to wartość funkcji określonej po lewej stronie równości, zaś \(\displaystyle{ P(k)}\) wartości po prawej stronie równości.
To drobiazg, ale te oznaczenia są do bani - ani lewa, ani prawa strona nie zależą od \(\displaystyle{ k}\).
VirtualUser pisze:Na tej podstawie stwierdzam, że \(\displaystyle{ L(k) \le P(k)}\)
To nie jest formalny dowód, tylko opowiadanie bajek na zasadzie "no wicie-rozumicie, jedno rośnie, drugie może też, ale nie za bardzo, więc na pewno nierówność jest prawdziwa". I nawet jeśli same obserwacje są słuszne, to ich opowiadanie nie stanowi dowodu.

JK
Awatar użytkownika
VirtualUser
Użytkownik
Użytkownik
Posty: 443
Rejestracja: 2 wrz 2017, o 11:13
Płeć: Mężczyzna
Podziękował: 113 razy
Pomógł: 15 razy

Re: Czy ten dowód jest dobry? Nierówność sumy modułów

Post autor: VirtualUser »

To nie jest elegancki dowód, z tym się zgadzam, ale proszę wskazać błąd w rozumowaniu. Nie twierdzę, że go tam nie ma, chciałbym go istotnie dojrzeć, tylko z pełnym szacunkiem, ale bez przyrównania dowodzenia do "opowiadania bajek".
Awatar użytkownika
leg14
Użytkownik
Użytkownik
Posty: 3132
Rejestracja: 5 lis 2014, o 20:24
Płeć: Mężczyzna
Lokalizacja: Radom
Podziękował: 154 razy
Pomógł: 475 razy

Czy ten dowód jest dobry? Nierówność sumy modułów

Post autor: leg14 »

No tak ale co to jest za moduł - na liczbach rzeczywistych zwykła wartośc bezwzględna ? Moduł liczb zepsolonych?
Awatar użytkownika
VirtualUser
Użytkownik
Użytkownik
Posty: 443
Rejestracja: 2 wrz 2017, o 11:13
Płeć: Mężczyzna
Podziękował: 113 razy
Pomógł: 15 razy

Re: Czy ten dowód jest dobry? Nierówność sumy modułów

Post autor: VirtualUser »

Moduł na liczbach rzeczywistych, wybacz, że o tym nie wspomniałem. Jeszcze nie wgryzłem się w zespolone.
Awatar użytkownika
leg14
Użytkownik
Użytkownik
Posty: 3132
Rejestracja: 5 lis 2014, o 20:24
Płeć: Mężczyzna
Lokalizacja: Radom
Podziękował: 154 razy
Pomógł: 475 razy

Re: Czy ten dowód jest dobry? Nierówność sumy modułów

Post autor: leg14 »

Zauważmy ponadto, że L(k+1) może być zarówno mniejsze/większe bądź równe L(k) gdyż z prawa trychotomii dodajemy wartość dodatnią/ujemną/zero, zaś ta zależnie od obecnego stanu L(k)sprawia, że L(k+1) >/</= L(k)
Zauważmy ponadto, że w obu przypadkach operujemy na tych samych wartościach elementów ze zbioru X.
prawda - ale tu nie ma żadnej substancji.
Na tej podstawie stwierdzam, że L(k) le P(k) zatem
Na podstawie czego konkretnie?

Weźmy dwie funkcje \(\displaystyle{ f,g}\) niech \(\displaystyle{ g}\) ma tę własność, że \(\displaystyle{ g(x_1+...x_{n+1}) \ge g(x_1+...+x_n)}\) dla dowolnych liczb rzeczywistych.
Zastosuj swój dowód do wykazania nierówności:
\(\displaystyle{ f(x_1+...+x_n) \le g(x_1 +...+x_n)}\).

Wszystko pasuje nieprawdaż? A wynikaq to z tego, że w dowodze nie skorzystałeś z definicji modułu
(zabawne, że niedawano podobny błąd zrobił Atiyah)
Jan Kraszewski
Administrator
Administrator
Posty: 34287
Rejestracja: 20 mar 2006, o 21:54
Płeć: Mężczyzna
Lokalizacja: Wrocław
Podziękował: 3 razy
Pomógł: 5203 razy

Re: Czy ten dowód jest dobry? Nierówność sumy modułów

Post autor: Jan Kraszewski »

VirtualUser pisze:To nie jest elegancki dowód, z tym się zgadzam, ale proszę wskazać błąd w rozumowaniu. Nie twierdzę, że go tam nie ma, chciałbym go istotnie dojrzeć, tylko z pełnym szacunkiem, ale bez przyrównania dowodzenia do "opowiadania bajek".
Nie myl "opowiadania bajek" z "opowiadaniem bzdur" - to co innego. "Opowiadanie bajek" oznacza, że nie ma tam dokładnego rozumowania, tylko dywagacje o swoich intuicjach.

Dowód to wnioskowanie, które wykonujesz w krokach, a każdy krok uzasadniasz. I wtedy można ocenić, czy takie rozumowanie jest poprawne, czy nie. U Ciebie nie ma opisanego rozumowania, tylko garść spostrzeżeń zakończonych autorytatywnym stwierdzeniem "z tego wynika teza". Nie jest to zatem dowód.

JK
Awatar użytkownika
leg14
Użytkownik
Użytkownik
Posty: 3132
Rejestracja: 5 lis 2014, o 20:24
Płeć: Mężczyzna
Lokalizacja: Radom
Podziękował: 154 razy
Pomógł: 475 razy

Czy ten dowód jest dobry? Nierówność sumy modułów

Post autor: leg14 »

Edit: przperaszam do pupy ta analogia, niech \(\displaystyle{ g, f}\) będą takie,że \(\displaystyle{ g \ge f \ge 0}\).
Zastpsuj swój dowód do wykazania, że
\(\displaystyle{ f(x_1 + ...+x_n) \le g(x_1) +...+g(x_n)}\)
Awatar użytkownika
VirtualUser
Użytkownik
Użytkownik
Posty: 443
Rejestracja: 2 wrz 2017, o 11:13
Płeć: Mężczyzna
Podziękował: 113 razy
Pomógł: 15 razy

Czy ten dowód jest dobry? Nierówność sumy modułów

Post autor: VirtualUser »

leg14 pisze:Edit: przperaszam do pupy ta analogia, niech \(\displaystyle{ g, f}\) będą takie,że \(\displaystyle{ g \ge f \ge 0}\).
Zastpsuj swój dowód do wykazania, że
\(\displaystyle{ f(x_1 + ...+x_n) \le g(x_1) +...+g(x_n)}\)
Spróbuję, tylko jeszcze się dopytam, co oznacza, że \(\displaystyle{ g \ge f \ge 0}\)? Dla każdego argumentu \(\displaystyle{ f}\) przyjmuje mniejszą lub równą wartość co \(\displaystyle{ g}\) dla tego samego?
Awatar użytkownika
leg14
Użytkownik
Użytkownik
Posty: 3132
Rejestracja: 5 lis 2014, o 20:24
Płeć: Mężczyzna
Lokalizacja: Radom
Podziękował: 154 razy
Pomógł: 475 razy

Czy ten dowód jest dobry? Nierówność sumy modułów

Post autor: leg14 »

Tak
ODPOWIEDZ